Is Integration by parts the way to go?

Calculus Level 4

0 1 x 50 ( ln x ) 150 d x \int_0^1 x^{50} (\ln x)^{150} \, dx

If the value of the integral above is equal to

A ! B C , \dfrac{A!}{B^C},

where A , B , A,B, and C C are positive integers, find the value of A + B + C A+B+C .


Bonus : Generalize 0 1 x m ( ln x ) n d x \displaystyle \int_0^1 x^{m} (\ln x)^{n} \, dx .


The answer is 352.

This section requires Javascript.
You are seeing this because something didn't load right. We suggest you, (a) try refreshing the page, (b) enabling javascript if it is disabled on your browser and, finally, (c) loading the non-javascript version of this page . We're sorry about the hassle.

4 solutions

We know that,

0 1 x a d x = 1 a + 1 \displaystyle \int_{0}^{1}x^{a}dx = \dfrac{1}{a+1}

Differentiating under the integral sign with respect to a,

0 1 x a l o g e ( x ) d x = 1 ( a + 1 ) 2 \displaystyle \int_{0}^{1}x^{a}log_{e}(x)dx = \dfrac{-1}{(a+1)^2}

Differentiating again with respect to a,

0 1 x a l o g e 2 ( x ) d x = 2 ( a + 1 ) 3 \displaystyle \int_{0}^{1}x^{a}log_{e}^{2}(x)dx = \dfrac{2}{(a+1)^3}

Repeating the process,

0 1 x a l o g e 3 ( x ) d x = 6 ( a + 1 ) 4 \displaystyle \int_{0}^{1}x^{a}log_{e}^{3}(x)dx = \dfrac{-6}{(a+1)^4}

We notice a pattern that,

0 1 x a l o g e n ( x ) d x = ( 1 ) n × n ! ( a + 1 ) n + 1 \displaystyle \int_{0}^{1}x^{a}log_{e}^{n}(x)dx = \dfrac{(-1)^{n} \times n!}{(a+1)^{n+1}}

For the given question, put n = 150 150 , a = 50 50 ,

\therefore I = 0 1 x 50 l o g e 150 d x = ( 1 ) 150 × 150 ! ( 51 ) 151 = 150 ! 5 1 151 \displaystyle \int_{0}^{1}x^{50}log_{e}^{150}dx = \dfrac{(-1)^{150} \times 150!}{(51)^{151 }}= \dfrac{150!}{51^{151}}

Comparing, we get, A = 150, B = 51 C = 151

A + B + C = 352 \therefore A + B + C = 352

Although I did by gamma function , your method was awesome.

Aakash Khandelwal - 4 years, 10 months ago

What is the source of this problem?

A Former Brilliant Member - 4 years, 10 months ago
Aditya Kumar
Jan 2, 2016

Alternate Solution:

Substitute x = e t 51 \large x=e^{\frac{-t}{51}}

Then the integral becomes:

1 51 151 0 e t t 150 d t = Γ ( 151 ) 51 151 = 150 ! 51 151 \large \frac { 1 }{ { 51 }^{ 151 } } \int _{ 0 }^{ \infty }{ { e }^{ -t } } { t }^{ 150 }dt\\ \large =\frac { \Gamma \left( 151 \right) }{ { 51 }^{ 151 } } \\ \large =\frac { 150! }{ { 51 }^{ 151 } }

@Aditya Kumar how did you go about substituting e^-t/51

Mardokay Mosazghi - 5 years, 5 months ago

Log in to reply

See first while solving I substituted x=e^t. Then I substituted t=-p/51 to get it to a form of gamma function. Hence I got an idea to directly substitute x=e^-t/51.

Aditya Kumar - 5 years, 5 months ago

Log in to reply

I think you mean e t 51 e^{\dfrac{-t}{51}} on the last line.

A Former Brilliant Member - 5 years, 5 months ago

Log in to reply

@A Former Brilliant Member Yes. Sorry for typo

Aditya Kumar - 5 years, 5 months ago

did same way . :)

Pawan pal - 4 years, 11 months ago
Christopher Boo
Jul 9, 2016

Let

f ( m , n ) = 0 1 x m ( ln x ) n = 1 m + 1 x m + 1 ( ln x ) n 0 1 n m + 1 0 1 x m ( ln x ) n 1 (Integration by Parts) = n m + 1 f ( m , n 1 ) = ( 1 ) n n m + 1 n 1 m + 1 n 2 m + 1 1 m + 1 f ( m , 0 ) = ( 1 ) n n m + 1 n 1 m + 1 n 2 m + 1 1 m + 1 1 m + 1 = ( 1 ) n n ! ( m + 1 ) n + 1 \begin{aligned} f(m,n) &= \int_0 ^ 1 x^m(\ln x)^n \\ &= \frac{1}{m+1}x^{m+1}(\ln x)^n \Bigg | ^1 _0 - \frac{n}{m+1}\int_0 ^1 x^m(\ln x)^{n-1} \quad \text{(Integration by Parts)}\\ &= -\frac{n}{m+1} f(m,n-1) \\ &= (-1)^n \frac{n}{m+1} \cdot \frac{n-1}{m+1} \cdot \frac{n-2}{m+1} \cdot \dots \cdot \frac{1}{m+1} \cdot f(m,0) \\ &= (-1)^n \frac{n}{m+1} \cdot \frac{n-1}{m+1} \cdot \frac{n-2}{m+1} \cdot \dots \cdot \frac{1}{m+1} \cdot \frac{1}{m+1} \\ &= \frac{(-1)^n\quad n!}{(m+1)^{n+1}} \end{aligned}

Well done!

Akshat Sharda - 3 years, 9 months ago

Easily one with a simple thinking!

Deepak Kumar - 4 years, 5 months ago

Notice the problem. The integral contains ln x \ln x and x x . If we put x = e t x = e^{-t} , we get t -t and e t e^{-t} . Terms present in Gamma. Thus let's get going. Introduction, Γ ( n + 1 ) = 0 e t t n d t Γ ( n Z + ) = ( n 1 ) ! \Gamma(n+1) = \int_0^\infty e^{-t}t^{n}\,dt\\\Gamma(n\in\mathbb Z^+) = (n-1)!

Substitute x = e t x = e^{-t} , d x = e t d t dx = -e^{-t} ~dt and ln x = t \ln x = -t I = 0 1 x m ( ln x ) n d x I = \int_0^1 x^m (\ln x)^n \,dx

Putting in the substitution, I = 0 e ( m + 1 ) t ( t ) n d t I = \int_0^\infty e^{-(m+1)t}\, (-t)^n \, dt

Again substituting ( m + 1 ) t = x (m+1)t = x , d x m + 1 = d t \dfrac{dx}{m+1} = dt . So, I = 0 e x ( x m + 1 ) n d x m + 1 = ( 1 ) n ( m + 1 ) n + 1 0 e x x n d x = ( 1 ) n Γ ( n + 1 ) ( m + 1 ) n + 1 \begin{aligned}I &= \int_0^\infty e^{-x} \left(\dfrac{-x}{m+1}\right)^n \dfrac{dx}{m+1}\\ &=\dfrac{(-1)^n}{(m+1)^{n+1}} \int_0^\infty e^{-x} x^n \, dx\\&= \dfrac{(-1)^n\Gamma(n+1)}{(m+1)^{n+1}} \end{aligned}

0 1 x m ( ln x ) n d x = ( 1 ) n Γ ( n + 1 ) ( m + 1 ) n + 1 \boxed{\displaystyle\int_0^1 x^m (\ln x)^n \,dx = \dfrac{(-1)^n\cdot\Gamma(n+1)}{(m+1)^{n+1}}}

Therefore, if n Z + n\in \mathbb Z^+ , 0 1 x m ( ln x ) n d x = ( 1 ) n n ! ( m + 1 ) n + 1 \int_0^1 x^m (\ln x)^n \,dx = \dfrac{(-1)^n\cdot n!}{(m+1)^{n+1}}

0 pending reports

×

Problem Loading...

Note Loading...

Set Loading...